You are on page 1of 22

𝑸𝑼𝑬𝑺𝑻𝑰𝑶𝑵 𝟏

On doit montrer que si la


lim 𝑎21𝑛+𝑖 = 8 𝑎𝑙𝑜𝑟𝑠 ∀𝜀 > 0, ∃𝑁 ∈ ℵ: ∀𝑛 ≥ 𝑁, |𝑎𝑛 − 8| < 𝜀
𝑛→∞

𝑆𝑜𝑖𝑡 𝜀 > 0
𝐶𝑜𝑚𝑚𝑒 lim 𝑎21𝑛+𝑖 = 8 : 𝑖𝑙 𝑒𝑥𝑖𝑠𝑡𝑒 𝑢𝑛 𝑟𝑎𝑛𝑔 𝑁O ∈ ℵ /∀ 𝑝 ≥ 𝑁𝑂
𝑛→∞
|𝑎21𝑝+𝑖 − 8| < 𝜀
𝑃𝑜𝑠𝑜𝑛𝑠 𝑁 = 𝑁𝑂 + 𝑖
𝑆𝑜𝑖𝑡 𝑛 ≥ 𝑁 : 𝑒𝑛 𝑓𝑖𝑥𝑎𝑛𝑡 𝑛 = 21𝑝 + 𝑖, 𝑜𝑛 𝑎 ∶ 21𝑝 + 𝑖 ≥ 21 𝑁0 +
𝑖, 𝑑𝑜𝑛𝑐 𝑝 ≥ 𝑁O, 𝑎𝑙𝑜𝑟𝑠 |𝑎2𝑃+𝑖 − 8| < 𝜀
𝐷′𝑜𝑢 ∀𝜀 > 0, ∃𝑁 ∈ ℵ : ∀𝑛 ≥ 𝑁, |𝑎𝑛 − 𝐵| < 𝜖
𝑃𝑎𝑟 𝑑𝑒𝑓𝑖𝑛𝑖𝑡𝑖𝑜𝑛, (𝑎𝑛 )𝑛 ≥ 0 𝑐𝑜𝑛𝑣𝑒𝑟𝑔𝑒 𝑣𝑒𝑟𝑠 𝐵.
𝑸𝑼𝑬𝑺𝑻𝑰𝑶𝑵 𝟐
A

C
𝑫𝑩 = ̅̅̅̅
̅̅̅̅̅ 𝑨𝑪 = 𝒅
̅̅̅̅̅
𝑶𝑯 = 𝒉
̅̅̅̅
𝑶𝑪 = ̅̅̅̅
𝑫𝑪 = 𝒃
𝟑𝟔𝟎 𝟏𝟖𝟎
̂=
𝒐 =
𝟐𝒏 𝒏

− − −> 𝐏𝐨𝐮𝐫 𝐧 𝐩𝐚𝐢𝐫

1 1
𝐴𝑛 = ×𝑛×𝑏×ℎ = ×𝑛×𝑏×ℎ
2 2
𝑏
180
sin ( )= 2
𝑛 𝑑
2
𝑏
=
𝑑
180
 𝑏 = 𝑑 sin ( )
𝑛
180 𝑑
 ℎ = cos ( )2
𝑛

1 180 180 𝑑
𝐴𝑛 = 𝑛𝑑 sin ( ) cos ( )
2 𝑛 𝑛 2
1 2𝑛 180 180
= × × 𝑑2 sin ( ) cos ( )
4 2 𝑛 𝑛
𝟏 𝟐 𝟑𝟔𝟎
𝑨𝒏 = 𝒏𝒅 𝐬𝐢𝐧 ( )
𝟖 𝒏

− − −> 𝐏𝐨𝐮𝐫 𝐧 𝐢𝐦𝐩𝐚𝐢𝐫


𝑐
𝜋
tan 𝐴̂ = tan ( ) = 2
(𝑟+ℎ)
2𝑛
𝜋
 𝑐 = 2(𝑟 + ℎ) tan ( )
2𝑛

𝜋 (𝑟 + ℎ) 𝜋 𝜋
Or cos ( )= → (𝑟 + ℎ) = 𝑑 cos ( ) tan ( )
2𝑛 𝑑 2𝑛 2𝑛
𝜋
= 2𝑑 sin ( )
2𝑛
̅̅̅̅̅
𝑂𝐻 ℎ 𝜋
Cos 𝑂̂ = ̅̅̅̅
= → ℎ = cos ( ) 𝑟
𝑂𝐵 𝑟 𝑛
𝑑
𝑑
Or cos 𝐴̂ = 2
→𝑟= 𝜋
𝑟 2 cos(2𝑛)
𝜋 𝑑
 ℎ = cos ( ) 𝜋
𝑛 2 cos(2𝑛 )

𝑛 𝜋 𝜋 𝑑
𝐴𝑛 = [(2𝑑 sin ( )) (cos ( ) )]
2 2𝑛 2𝑛 2 cos ( 𝜋 )
2𝑛
𝜋
𝑛𝑑2 sin(2𝑛) 𝜋
= [ 𝜋 cos ( )]
2 cos(2𝑛) 𝑛

𝑛𝑑2 𝜋 𝜋
= [tan ( ) cos ( )]
2 2𝑛 𝑛

Nous savons que cos 2𝑥 = 2 cos2 𝑥 − 1


𝑛𝑑2 𝜋 2𝜋
 𝐴𝑛 = [tan ( ) cos ( )]
2 2𝑛 2𝑛

𝑛𝑑2 𝜋 𝜋
= [tan ( ) [2 cos2 ( ) − 1]]
2 2𝑛 2𝑛

𝑛𝑑2 𝜋 𝜋 𝜋
= [tan ( ) 2 cos2 ( ) − tan ( )]
2 2𝑛 2𝑛 2𝑛
𝜋
𝑛𝑑2 sin(2𝑛) 𝜋 𝜋
= [ 𝜋 2 cos2 ( ) − tan ( )]
2 cos(2𝑛) 2𝑛 2𝑛

𝑛𝑑2 𝜋 𝜋 𝜋
= [2 sin ( ) cos ( ) − tan ( )]
2 2𝑛 2𝑛 2𝑛

- sin 2𝑥 = 2 sin 𝑥 cos 𝑥


𝑛𝑑2 2𝜋 𝜋
 𝐴𝑛 = [sin ( ) − tan ( )]
2 2𝑛 2𝑛
𝑛𝑑2 𝜋 𝜋
𝐴𝑛 = [sin ( ) − tan ( )]
2 𝑛 2𝑛

𝑃𝑜𝑢𝑟 𝑑 = 1
𝒏 𝝅 𝝅
𝑨𝒏 = [𝐬𝐢𝐧 ( ) − 𝐭𝐚𝐧 ( )]
𝟐 𝒏 𝟐𝒏

𝐸𝑛 𝑐𝑎𝑙𝑐𝑢𝑙𝑎𝑛𝑡, 𝑜𝑛 𝑡𝑟𝑜𝑢𝑣𝑒 𝑙𝑒𝑠 𝑣𝑎𝑙𝑒𝑢𝑟𝑠 𝑐𝑖 𝑎𝑝𝑟𝑒𝑠 𝑝𝑜𝑢𝑟:


𝐴11 = 0.75874844565551
𝐴12 = 0.7499999999999
𝐴13 = 0.766308926924432
𝐴14 = 0.7592965434557267
𝐴15 = 0.771055916640216
𝐴16 = 0.7653668647301796
𝐴17 = 0.7742299083835948
𝐴18 = 0.7695453224827546
𝐴19 = 0.7764561643846877
𝐴20 = 0.7725424859373685
𝑸𝒖𝒆𝒔𝒕𝒊𝒐𝒏 𝟐𝒃𝟏

𝑛
∀𝑛 ≥ 1, 𝐴2𝑛 = 𝑠𝑖𝑛(𝜋𝑛)
4

𝑆𝑎𝑐ℎ𝑎𝑛𝑡 𝑞𝑢𝑒 𝑛 ≥ 2
𝑂𝑛 𝑎: 0 ≤ 2 ≤ 𝑛
1 1 𝜋 𝜋
0< ≤ ;0< ≤
𝑛 2 𝑛 2
𝜋
Etant donné que La fonction sinus est croissant sur [0, ],
2
𝜋 𝜋
𝑆𝑖𝑛𝑢𝑠 0 < 𝑠𝑖𝑛𝑢𝑠 < 𝑠𝑖𝑛𝑢𝑠
𝑛 2
𝜋
𝐷𝑜𝑛𝑐: 0 < 𝑠𝑖𝑛𝑢𝑠 ≤ 1
𝑛
𝑛 𝜋 𝑛
0 < 𝑠𝑖𝑛𝑢𝑠 ≤ (1)
4 𝑛 4
𝑛 𝜋
𝑂𝑛 𝑟𝑒𝑚𝑎𝑟𝑞𝑢𝑒 𝑑′𝑎𝑝𝑟𝑒𝑠 (1) 𝑞𝑢𝑒 ∀𝑛 ≥ 2, 𝐴2𝑛 = 𝑠𝑖𝑛𝑢𝑠 > 0
4 𝑛

𝐼𝑛𝑡𝑟𝑜𝑑𝑢𝑖𝑠𝑜𝑛𝑠 𝑙𝑎 𝑓𝑜𝑛𝑐𝑡𝑖𝑜𝑛 Ϝ:
]0 + ∞[→ ℝ
Ϝ :{ 𝑥 𝜋
𝑥 → 𝑠𝑖𝑛
4 𝑥

𝑥 ]0 2 +∞

𝐹(𝑥) 0 1 𝜋
+ +
2 4

Ϝ(𝑥)𝑥→0 → 0
𝜋
Ϝ(𝑥)𝑥→∞ →
4
𝜋
𝑂𝑛 𝑎𝑢𝑟𝑎 ∶ ∀𝑥 𝜖]0, +∞[ , 𝑓(𝑥) 𝜖]0, [ ⊂ ]0, +∞[
4
𝜋
𝐶𝑒𝑐𝑖 𝑝𝑟𝑜𝑢𝑣𝑒 𝑞𝑢𝑒 ]0, [ 𝑒𝑠𝑡 𝑠𝑡𝑎𝑏𝑙𝑒 𝑝𝑎𝑟 𝐹
4
𝜋
𝐴𝑙𝑜𝑟𝑠 ∀𝑥 > 0, 0 < Ϝ(𝑥) <
4
[2, +∞[→ ℝ
𝐺: { 𝑛 𝜋
𝑛 → 𝐴2𝑛 = 𝑆𝑖𝑛( ),
4 𝑛

𝐺 𝑒𝑠𝑡 𝑢𝑛𝑒 𝑟𝑒𝑠𝑡𝑟𝑖𝑐𝑡𝑖𝑜𝑛 𝑑𝑒 𝑓.


1 𝜋
𝐷𝑜𝑛𝑐 ∀𝑛 ≥ 2, < 𝑔(𝑛) = 𝐴𝑛 < 𝐴𝑛 𝑒𝑠𝑡 𝑏𝑜𝑟𝑛𝑒𝑒.
2 4

𝑸𝒖𝒆𝒔𝒕𝒊𝒐𝒏 𝟐𝒃𝟐

2𝑛−1 𝜋 𝜋
∀𝑛 ≥ 2, 𝐴𝑛−1 = (𝑆𝑖𝑛2𝑛−1 − 𝑡𝑔2𝑛−2)
2

𝑃𝑎𝑟 𝑎𝑛𝑎𝑙𝑜𝑔𝑖𝑒 𝑎𝑏𝑒𝑐 𝑏1, 𝑜𝑛 𝑎 ∶


0 < 2 < 𝑛 < 2𝑛 → −1 < 1 ≤ 𝑛 − 1 < 2𝑛 − 1 < 4𝑛 − 2
1 1
𝐷𝑜𝑛𝑐: 0 < < <1
4𝑛−2 2𝑛−1
𝜋 𝜋 𝜋
0< < <
4𝑛−2 2𝑛−1 2
𝜋 𝜋
𝑆𝑖𝑛 < 𝑆𝑖𝑛
4𝑛−2 2𝑛−1
𝜋 𝜋
𝐶𝑜𝑚𝑚𝑒 ∀𝑛 ≥ 2 𝑆𝑖𝑛( ) > 0 𝑒𝑡 𝐶𝑜𝑠( ) > 0, 𝑜𝑛 𝑎 ∶
4𝑛−2 4𝑛−2
𝜋 𝜋
Sin(4𝑛−2) Sin(2𝑛−1) 𝜋
𝜋 < 𝜋 < 𝑆𝑖𝑛( )
Cos(4𝑛−2) Cos(4𝑛−2) 2𝑛−2
𝜋
𝜋 sin(2𝑛−1) 𝜋
𝐷𝑜𝑛𝑐 𝑡𝑔( )< 𝜋 < 𝑆𝑖𝑛( )
4𝑛−2 cos(4𝑛−2) 2𝑛−2

𝜋 𝜋
𝑂𝑛 𝑎 ∶ 𝑡𝑔( ) < Sin( )
4𝑛−2 2𝑛−2
𝜋 𝜋
𝐷𝑜𝑛𝑐 𝑆𝑖𝑛( ) −𝑡𝑔( ) >0
2𝑛−2 4𝑛−2
𝐶𝑜𝑚𝑚𝑒 𝑛 ≥ 2, 𝑜𝑛 𝑎 ∶ 2𝑛 ≥ 4 > 1
𝐷𝑜𝑛𝑐 2𝑛 > 1, 2𝑛 − 1 > 0
2𝑛−1 𝜋 𝜋
𝑂𝑛 𝑎 𝑑𝑜𝑛𝑐 ∶ ∀𝑛 ≥ 2, 𝑆𝑖𝑛( ) −𝑡𝑔( )>0 (2)
2 2𝑛−2 4𝑛−2

𝑫′𝒂𝒑𝒓𝒆𝒔 (𝟐), ∀𝑛 ≥ 2, 𝐴2𝑛−1 > 0


]0, +∞[→ ℝ
Ϝ :{ 2𝑥−1 𝜋 𝜋
𝑥→ (sin − tg )
2 2𝑛−2 4𝑛−2

1 𝜋 1 𝜋
lim 𝑓(𝑥) = lim (2𝑥 − 1) 𝑆𝑖𝑛 - lim (2𝑥 − 1) 𝑡𝑔
𝑥→0 2 𝑥→0 2𝑛−1 2 𝑥→0 2(2𝑥−1)
𝜋 𝜋
𝜋 𝑠𝑖𝑛2𝑥−1 𝑡𝑔2(2𝑥−1)
= lim 𝜋 - 𝜋lim 𝜋
2 𝑥→0 2𝑥−1 𝑥→0 2(2𝑥−1)
𝜋
= 𝑥0 − 𝜋𝑥0 = 0
2

𝐷𝑜𝑛𝑐 𝑙𝑖𝑚𝑓 (𝑥 ) = 0
𝜋
,lim 𝑓(𝑥) =
𝑥→0 4

𝑥 0 +∞
𝜋
𝐹(𝑥) 0 +
4

𝜋
𝑂𝑛 𝑎: ∀ 𝑥 𝜖]0, +∞[, 𝑓(𝑋) 𝜖 ]0, [⊂]0, +∞[
4
𝜋
𝐷𝑜𝑛𝑐 ]0, [𝑒𝑠𝑡 𝑠𝑡𝑎𝑏𝑙𝑒 𝑝𝑜𝑢𝑟 𝑓.[
4
𝜋
𝐴𝑖𝑛𝑠𝑖, ∀ 𝑥 𝜖]0, +∞[, 0 < 𝑓(𝑋) <
4

𝐺: [2, +∞[→ ℝ
𝑥 → 𝑓(𝑥 ) 𝑒𝑠𝑡 𝑢𝑛𝑒 𝑟𝑒𝑠𝑡𝑟𝑖𝑐𝑡𝑖𝑜𝑛 𝑑𝑒 𝑓
𝜋
𝐷𝑜𝑛𝑐 ∀𝑛 ≥ 2, 0< 𝑔(𝑋) <
4
𝜋
𝐷′𝑜𝑢 ∀𝑛 ≥ 2, 0< 𝐴2𝑛−1 <
4

𝐴2𝑛−1 𝑒𝑠𝑡 𝑏𝑜𝑟𝑛𝑒𝑒.

𝑸𝒖𝒆𝒔𝒕𝒊𝒐𝒏 𝟐𝒄
𝑛 𝜋
∀ 𝑛 ≥ 2, 𝐴2𝑛 = ∗ 𝑆𝑖𝑛
2 𝑛

𝜋
[2, +∞[ → ]0, [
4
.f: { 𝑥 𝜋
𝑥 → ∗ 𝑆𝑖𝑛
2 𝑥
1 𝜋 𝜋 𝜋
*f’(x) = 𝑆𝑖𝑛 − 𝐶𝑜𝑠
2 𝑥 2𝑥 𝑥
1 𝜋 𝜋 𝜋
= (𝑆𝑖𝑛 − 𝐶𝑜𝑠 ) > ? 0
2 𝑥 𝑥 𝑥

.∀ 𝑛 ≥ 2, 𝑇𝑔𝑥 > 𝑥 > 𝑆𝑖𝑛𝑥


1 1 1
* Donc, 𝑆𝑖𝑛 > > 𝑡𝑔
𝑥 𝑥 𝑥
𝜋 𝜋 𝜋
𝑆𝑖𝑛 > > 𝑡𝑔
𝑥 𝑥 𝑥
𝜋 𝜋
* On a: 𝑆𝑖𝑛 > (3)
𝑥 𝑥
𝜋 𝜋
De plus, ∀ 𝑥 ≥ 4, 1 > 𝐶𝑜𝑠 ≥ 𝑆𝑖𝑛
𝑥 𝑥
𝜋 𝜋
𝜋 > 𝜋𝐶𝑜𝑠 ≥ 𝑆𝑖𝑛
𝑥 𝑥
𝜋 𝜋 𝜋 1 𝜋
> 𝐶𝑜𝑠 ≥ 𝑆𝑖𝑛
𝑥 𝑥 𝑥 𝑥 𝑥
𝜋 𝜋 𝜋 𝜋 1 𝜋
𝑆𝑖𝑛 > > 𝐶𝑜𝑠 > 𝑆𝑖𝑛
𝑥 𝑥 𝑥 𝑥 𝑥 𝑥
𝜋 𝜋 𝜋
* On a ∶ 𝑆𝑖𝑛 > 𝐶𝑜𝑠
𝑥 𝑥 𝑥
𝜋 𝜋 𝜋
* Donc ∀ 𝑥 ≥ 4, 𝑆𝑖𝑛 − 𝐶𝑜𝑠 >0
𝑥 𝑥 𝑥
1 𝜋 𝜋 𝜋
* On a: (𝑆𝑖𝑛 − 𝐶𝑜𝑠 ) > 0
2 𝑥 𝑥 𝑥
′(
*𝑓 𝑥 ) > 0 ∀ 𝑥 > 4, 𝑓𝑒𝑠𝑡 𝑐𝑟𝑜𝑖𝑠𝑠𝑎𝑛𝑡𝑒.

*𝐷 𝑜𝑢 ∀𝑛 > 2, 𝐴2𝑛 𝑒𝑠𝑡 𝑐𝑟𝑜𝑖𝑠𝑠𝑎𝑛𝑡𝑒

𝑸𝒖𝒆𝒔𝒕𝒊𝒐𝒏 𝟐𝒅
𝑺𝒆𝒍𝒐𝒏 𝒍𝒆 𝒕𝒉𝒆𝒐𝒓𝒆𝒎𝒆 𝒅𝒆𝒔 𝒔𝒖𝒊𝒕𝒆𝒔 𝒎𝒐𝒏𝒐𝒕𝒐𝒏𝒆𝒔, 𝒖𝒏𝒆 𝒔𝒖𝒊𝒕𝒆 𝒆𝒔𝒕 𝒄𝒐𝒏𝒗𝒆𝒓𝒈𝒆𝒏𝒕𝒆
𝒔𝒔𝒊 𝒆𝒍𝒍𝒆 𝒆𝒔𝒕 𝒎𝒐𝒏𝒐𝒕𝒐𝒏𝒆 𝒆𝒕 𝒃𝒐𝒓𝒏𝒆𝒆 .
𝑶𝒓 𝒅′𝒂𝒑𝒓𝒆𝒔 𝒃 𝒆𝒕 𝒄, (𝑨𝟐𝒏 ) 𝒏 ≥ 𝟐 𝒆𝒕 (𝑨𝟐𝒏−𝟏 ) 𝒏
≥ 𝟐 𝒔𝒐𝒏𝒕 𝒎𝒐𝒏𝒐𝒕𝒐𝒏𝒆𝒔 𝒆𝒕 𝒃𝒐𝒓𝒏𝒆𝒆𝒔.
𝑫𝒐𝒏𝒄 𝒆𝒍𝒍𝒆𝒔 𝒔𝒐𝒏𝒕 𝒄𝒐𝒏𝒗𝒆𝒓𝒈𝒆𝒏𝒕𝒆𝒔.
𝝅
𝐥𝐢𝐦 𝑨𝟐𝒏 =
𝒏→∞ 𝟒

𝝅
𝐥𝐢𝐦 𝑨𝟐𝒏−𝟏 =
𝒏→∞ 𝟒
𝝅
𝑫𝒐𝒏𝒄 𝒍𝒂 𝒔𝒖𝒊𝒕𝒆 (𝑨𝒏 ) 𝒏 ≥ 𝟑 𝒄𝒐𝒏𝒗𝒆𝒓𝒈𝒆 𝒆𝒕 𝒔𝒂 𝐥𝐢𝐦 𝑨𝒏 =
𝒏→∞ 𝟒
𝑸𝑼𝑬𝑺𝑻𝑰𝑶𝑵 𝟑

𝑆𝑜𝑖𝑡 𝑙𝑎 𝑠𝑢𝑖𝑡𝑒 (𝑎𝑛 ) 𝑑𝑒𝑓𝑖𝑛𝑖𝑒 𝑝𝑎𝑟 𝑎1 = 16 𝑒𝑡 𝑝𝑜𝑢𝑟 𝑡𝑜𝑢𝑡 𝑛 ≥ 1, 𝑎𝑛+1 =


1 + ∏𝑛𝑘=1 𝑎k
1
𝐶𝑎𝑙𝑐𝑢𝑙𝑒𝑟 ∑∞
𝑛=1 𝑎𝑛

𝑆𝑜𝑙𝑢𝑡𝑖𝑜𝑛
𝑃𝑜𝑢𝑟 𝑡𝑜𝑢𝑡 𝑛 ≥ 1 ; 𝑎𝑛+1 = 1 + ∏𝑛𝑘=1 𝑎𝑘
= 1 + (∏𝑛−1
𝑘=1 𝑎𝑘 )𝑎𝑛

= 1 + (∏𝑛−1
𝑘=1 𝑎𝑘+1 − 1)

𝑆𝑖 𝑎𝑛+1 = 1 + ∏𝑛𝑘=1. 𝑎𝑘 , 𝑎𝑙𝑜𝑟𝑠 𝑎𝑛 = 1 + ∏𝑛−1


𝑘=1 𝑎

𝐸𝑡 𝑑𝑜𝑛𝑐 𝑎𝑛+1 = 1 + (𝑎𝑛 − 1)𝑎𝑛


𝐷𝑜𝑛𝑐 𝑎𝑛+1 − 1 = (𝑎𝑛 − 1)𝑎𝑛
𝑂𝑛 𝑎𝑢𝑟𝑎:
1 1 1
= −
𝑎𝑛+1 − 1 𝑎𝑛 − 1 𝑎𝑛
𝐷𝑜𝑛𝑐:
1 1 1
= −
𝑎𝑛 𝑎𝑛 − 1 𝑎𝑛+1 − 1
𝐷′𝑎𝑝𝑟𝑒𝑠 (1) 𝑙𝑎 𝑠𝑢𝑖𝑡𝑒 𝑎𝑛 𝑒𝑠𝑡 𝑑𝑒𝑓𝑖𝑛𝑖𝑒 𝑐𝑜𝑚𝑚𝑒 𝑠𝑢𝑖𝑡 ∶
𝑎1 = 16
𝑎𝑛 : {𝑎 = 𝑎𝑛2 − 𝑎𝑛+1 ∀ 𝑛 ≥ 1
𝑛+1

𝑅 + → 𝑅+∗
𝐸𝑛 𝑖𝑛𝑡𝑟𝑜𝑑𝑢𝑖𝑠𝑎𝑛𝑡 𝑢𝑛𝑒 𝑓𝑜𝑛𝑐𝑡𝑖𝑜𝑛 𝑓: {
𝑥 → 𝑥2 − 𝑥 + 1
𝑂𝑛 𝑎: 𝑎1 = 16 𝜖 𝑅∗+
𝑎𝑛+1 = 𝑓(𝑎𝑛 )
𝑓′(𝑥) = 2𝑥 − 1
𝑂𝑛 𝑎:
∀ 𝑥 ∈ 𝑅 + , 𝑓(𝑥 ) ∈ 𝑅+∗ 𝐶 𝑅 +
{𝐷𝑜𝑛𝑐 𝑅+∗ 𝑒𝑠𝑡 𝑠𝑡𝑎𝑏𝑙𝑒 𝑝𝑜𝑢𝑟 𝑓
16 ∈ 𝑅+∗

𝐷′𝑜𝑢 ∀ 𝑛 ≥ 1 , 𝑎𝑛 𝜖 [0, +∞[


𝐶𝑜𝑛𝑠𝑒𝑞𝑢𝑒𝑛𝑐𝑒: 𝑎𝑛 𝑛′𝑒𝑠𝑡 𝑝𝑎𝑠 𝑚𝑎𝑗𝑜𝑟𝑒𝑒.
1
𝐷𝑒 𝑝𝑙𝑢𝑠 ∀ 𝑥 ≥ , 𝑓(𝑥) − 𝑥 ≥ 0
2

𝐷𝑜𝑛𝑐 ∃ 𝑛0 ≥ 1 𝑡𝑒𝑙 𝑞𝑢𝑒 𝑎𝑛 𝑒𝑠𝑡 𝑐𝑟𝑜𝑖𝑠𝑠𝑎𝑛𝑡𝑒.


𝐶𝑜𝑚𝑚𝑒 𝑎𝑛 𝑒𝑠𝑡 𝑐𝑟𝑜𝑖𝑠𝑠𝑎𝑛𝑡𝑒 𝑒𝑡 𝑛𝑜𝑛 𝑚𝑎𝑗𝑜𝑟𝑒𝑒, 𝑑𝑜𝑛𝑐 𝑎𝑛 𝑑𝑖𝑣𝑒𝑟𝑔𝑒.

1
𝑂𝑛 𝑎 𝑑𝑜𝑛𝑐 lim 𝑎𝑛 = +∞ ; 𝑑𝑜𝑛𝑐 lim =0
𝑛→∝ 𝑛→∞ 𝑎𝑛
1 1
𝐶𝑜𝑛𝑠𝑒𝑞𝑢𝑒𝑛𝑐𝑒 lim =0 ( lim = 0)
𝑛→∞ 𝑎𝑛+1 𝑛→∞ 𝑎𝑛 −1
1
𝐴𝑣𝑒𝑐 𝑡𝑜𝑢𝑡 𝑐𝑒 𝑞𝑢′𝑜𝑛 𝑎 𝑑𝑒𝑗𝑎, 𝑐𝑎𝑙𝑐𝑢𝑙𝑜𝑛𝑠 𝑙𝑎 𝑠𝑜𝑚𝑚𝑒 ∶ ∑∞
𝑛=1 𝑎𝑛
1 1 1
𝑂𝑛 𝑎: ∑∞
𝑛=1 = ∑∞
𝑛=1 = lim ∑𝑛𝑘=1 = lim 𝑆𝑛
𝑎𝑛 𝑎𝑛 𝑛→∞ 𝑎𝑘 𝑛→∞
1
𝐴𝑣𝑒𝑐 𝑆𝑛 = ∑∞
𝑛=1 𝑎𝑘

Calcul de 𝑆𝑛 :
1 1 1
𝑆𝑛 = ∑∞
𝑛=1 = ∑𝑛𝑘=1 ( − )
𝑎𝑘 𝑎𝑘−1 𝑎𝑘+1 −1
1 1 1 1
= ∑𝑛𝑘=1 − ∑𝑛𝑘=1 = ∑𝑛𝑘=1 − ∑𝑛+1
𝑘=2
𝑎𝑘−1 𝑎𝑘+1 −1 𝑎𝑘−1 𝑎𝑘−1
1 1 1 1
= + ∑𝑛𝑘=2 − ∑𝑛𝑘=2 −
𝑎1 −1 𝑎𝑘−1 𝑎𝑘−1 𝑎𝑛+1 −1
1 1 1 1
𝑆𝑛 = − , donc Sn = −
𝑎1 −1 𝑎𝑛+1 −1 16−1 𝑎𝑛+1 −1
1 1 1
𝐴𝑖𝑛𝑠𝑖 ∑∞
𝑛=1 = lim 𝑆𝑛 = lim − lim
𝑎𝑛 𝑛→∞ 𝑛→∞ 𝐷−1 𝑛→∞ 𝑎𝑛+1 −1

0
1 1 1
𝐷𝑜𝑛𝑐 ∑∞
𝑛=1 = lim =
𝑎𝑛 𝑛→∞ 16−1 16−1

1 1
𝐷′𝑜𝑢 ∀ 𝑛 ≥ 1 , ∑∞
𝑛=1 =
𝑎𝑛 16−1

1 1 1
∀ 𝑛 ≥ 1, ∑∞
𝑛=1 = =
𝑎𝑛 16−1 15
𝑄𝑈𝐸𝑆𝑇𝐼𝑂𝑁 4
1 1
.𝑎) 𝑓′(𝑥) = +
(12+𝑥) (12−𝑥)
−1 1
.𝑓′′(𝑥) = +
(12+𝑥)2 (12−𝑥)2
2 2
.𝑓′′′(𝑥) = 3 +
(12+𝑥) (12−𝑥)3

(𝑛) (−1)(𝑛−1) (𝑛−1)! (𝑛−1)!


.𝑆𝑢𝑝𝑝𝑜𝑠𝑜𝑛𝑠 𝑞𝑢𝑒 ∀𝑛 ≥ 1 𝑓 (𝑥) = +
(12+𝑥)𝑛 (12−𝑥)𝑛

𝐸𝑡 𝑝𝑎𝑟 𝑟𝑒𝑐𝑐𝑢𝑟𝑒𝑛𝑐𝑒, 𝑚𝑜𝑛𝑡𝑟𝑜𝑛𝑠 𝑞𝑢𝑒 ∀𝑛 ≥ 1 𝑃(𝑛): 𝑓 (𝑛) (𝑥) =


(−1)(𝑛−1)! (𝑛−1)! (𝑛−1)!
+
(12+𝑥)𝑛 (12−𝑥)𝑛

𝐼𝑛𝑖𝑡𝑖𝑎𝑙𝑖𝑠𝑎𝑡𝑖𝑜𝑛:

1 1
𝑃𝑜𝑢𝑟 𝑛 = 1, 𝑓′(𝑥) = 1 +
(12+𝑥) (12−𝑥)1

(−1)(0) (0)! 0! 1 1
= + = +
(12+𝑥)1 (12−𝑥)1 (12+𝑥) (12−𝑥)
𝐷′𝑜𝑢 𝑃(1).
𝐻𝑒𝑟𝑒𝑑𝑖𝑡𝑒 ∶

𝑆𝑜𝑖𝑡 𝑛 ≥ 1, 𝑠𝑢𝑝𝑝𝑜𝑠𝑜𝑛𝑠 𝑃(𝑛) 𝑒𝑡 𝑚𝑜𝑛𝑡𝑟𝑜𝑛𝑠 𝑃(𝑛 + 1)

𝐸𝑛 𝑒𝑓𝑓𝑒𝑡 ∶ 𝑓 (𝑛+1) (𝑥) = [𝑓 (𝑛+1) (𝑥)]′


(−1)(𝑛−1) (𝑛 − 1)! (𝑛 − 1)!
=[ + ]′ 𝐷′𝑎𝑝𝑟𝑒𝑠 𝑃(𝑛)
(12 + 𝑥)𝑛 (12 − 𝑥)𝑛

(−1)𝑛−1 (𝑛−1)!(−𝑛) (𝑛−1)!(𝑛)


= +
(12+𝑥)𝑛+1 (12−𝑥)𝑛+1
(−1)𝑛−1 (−1).𝑛(𝑛−1)! 𝑛(𝑛−1)!
= +
(12+𝑥)𝑛+1 (12−𝑥)𝑛+1

(−1)𝑛 𝑛! 𝑛!
= 𝑛+1 + 𝐷′𝑜𝑢 𝑃(𝑛 + 1)
(12+𝑥) (12−𝑥)𝑛−1

(𝑛) (−1)(𝑛−1) (𝑛−1)! (𝑛−1)!


𝐶𝑜𝑛𝑐𝑙𝑢𝑠𝑖𝑜𝑛 ∶ ∀𝑛 ≥ 1 , 𝑓 (𝑥) = +
(12+𝑥)𝑛 (12−𝑥)𝑛

𝑇𝑟𝑜𝑢𝑣𝑜𝑛𝑠 𝑢𝑛 𝑝𝑜𝑖𝑛𝑡 𝑀 ≥ 0 𝑡𝑒𝑙 𝑞𝑢𝑒 𝑀 = 𝑠𝑢𝑝|𝑓 (𝑛) (𝑥)|

−156 156
𝑆𝑜𝑖𝑡 𝑛 ≥ 1, 𝑠𝑜𝑖𝑡 𝑥 𝜖 [ , ]
21 21

𝑀 = 𝑠𝑢𝑝|𝑓 (𝑛) (𝑥)| 𝐷𝑜𝑛𝑐 − 𝑀 ≤ 𝑓 (𝑛) (𝑥 ) ≤ 𝑀


(−1)(𝑛−1) (𝑛−1)! (𝑛−1)! −156 156
𝑆𝑜𝑖𝑡 − 𝑀 ≤ + ≤ 𝑀; 𝑥𝜖 [ , ] , 𝑜𝑛 𝑎:
(12+𝑥)𝑛 (12−𝑥)𝑛 21 21

−156 156
≤𝑥≤ 𝐸−𝐹 ≤𝐸+𝑥 ≤𝐸+𝐹 36
21 21
{−156 (1) ↔ { 96 136 𝑎𝑣𝑒𝑐 ≥
≤ −𝑥 ≤
156 ≤ 12 − 𝑥 ≤ 21
21 7
21 21
0:

1 1 1
36
( )𝑛 ≤ (12 + 𝑥) ≤ 𝑛 136
( )𝑛 136 ≤ ≤ 36
( 7 )𝑛 (12+𝑥)𝑛 (21)𝑛
{ 21
36
7
136 𝑛 ↔ { 1 1 1
( )𝑛 ≤ (12 + 𝑥)𝑛 ≤ ( ) 136 ≤ ≤ 36
21 7 ( 7 )𝑛 (12−𝑥)𝑛 (21)𝑛
(−1)(𝑛−1) (𝑛−1)! (𝑛−1)!
≤ 36 (1)′
(12+𝑥)𝑛 ( )𝑛
21
∀𝑛 ≥ 1 𝑜𝑛 𝑎 ∶ { (𝑛−1)! (𝑛−1)!
≤ 36 (2)′
(12−𝑥)𝑛 (21)𝑛

(−1)(𝑛−1) (𝑛−1)! (𝑛−1)! (𝑛−1)!


(1)′ + (2)′ ∶ + ≤ 2 36
(12+𝑥)𝑛 (12−𝑥)𝑛 (21)𝑛

(𝑛−1)!
𝐷𝑜𝑛𝑐 ∶ 𝑓 (𝑛) (𝑥) ≤ 2 36
(21)𝑛

(𝑛−1)!
𝑃𝑜𝑠𝑜𝑛𝑠 𝑀 = 2 36
(21)𝑛
(𝑛−1)! −156 156
𝐷′𝑜𝑢 𝑠𝑢𝑝|𝑓 (𝑛) (𝑥)| = 2 36 , ∀𝑛 ≥ 1, ∀ 𝑥 𝜖 [ , ]
( )𝑛 21 21
21

(𝑛) (−1)(𝑛−1) (𝑛−1)! (𝑛−1)! (𝑛−1)!(1−(−1)𝑛


a) 𝑓 (0) = + =
12𝑛 12𝑛 12𝑛

𝑓 (𝑘) (0)
𝑇(𝑥) = ∑𝑛𝑘=1 𝑥 𝑘 + 𝑅𝑛
𝑘!

(𝑘−1)!(1−(−1)𝑘 )
= ∑𝑛𝑘=1 𝑥 𝑘 + 𝑅𝑛
𝑘! 12𝑛

(1−(−1)𝑘 )
= ∑𝑛𝑘=1 𝑥 𝑘 + 𝑅𝑛
𝑘.12𝑛

𝑓 (𝑛+1) (𝜀)
𝐴𝑣𝑒𝑐 𝑅𝑛(𝑥) = 𝑥 𝑛+1
(𝑛+1)!
𝑥 𝑛+1 (−1)(𝑛−1) (𝑛−1)! (𝑛−1)!
= [ + ]
(𝑛+1)! (12+𝜀)𝑛 (12−𝜀)𝑛

𝑥 𝑛+1 ∗(𝑛−1)! (−1)(𝑛−1) 1


= [ + ]
(𝑛+1)! (12+𝜀)𝑛 (12−𝜀)𝑛

𝑥 𝑛+1 (−1)(𝑛−1) 1
= [ + ]
𝑛(𝑛+1) (12+𝜀)𝑛 (12−𝜀)𝑛

𝑀
𝑐) 𝑂𝑛 𝑠𝑎𝑖𝑡 𝑞𝑢𝑒 𝑅𝑛 ≤ 𝑋 𝑛+1
(𝑛+1)!

∀ |𝑓 (𝑛+1) (𝑥)| < 𝑀


2(𝑛−1)! 2(𝑛−1)!
𝑀= 156 = 96 𝑛
(12− )𝑛 (21)
21

𝑀
0 < 𝑅𝑛 < 𝑋 𝑛+1
(𝑛+1)!

2(𝑛−1)!𝑋 𝑛+1
0 < 𝑅𝑛 < 96 𝑛
(21) (𝑛+1)!

2(𝑛−1)!𝑋 𝑛+1
lim 𝑛
𝑛→∞ (96) (𝑛+1)!
21

2(𝑛−1)!𝑋 𝑛+1 𝑋 𝑛+1


lim 𝑛 [ lim ]
𝑛→∞ (96) (𝑛+1)! 𝑛→∞ (𝑛+1)!
21

2(𝑛−1)!𝑋 𝑛+1
𝐷′𝑜𝑢 0 < 𝑅𝑛 < 96 𝑛
(21) (𝑛+1)!

−156 156
𝑇(𝑥) = 𝑓(𝑥) ∀ 𝑥 𝜖 [ , ]
21 21
−156 156 (−1)𝑘−1 +1∗𝑥 𝑘
𝑑) ∀ 𝑥 𝜖 [ , ] , 𝑇𝑛(𝑥) = ∑𝑛𝑘=0
21 21 𝑘∗12𝑘
n
(k-1)! (1-(-1)k ) k
Tn(x) = ∑ * *x
k! 12k
k=0

x n+1 (1-(-1)n+1 )
|rn | = | * n+1
| < 10-4
(n + 1)! 12
|x|n+1 1
On a: |rn | = * n+1 *|(1-(-1)n+1 )|
(n + 1)! 12
0 si n est impair (1)
={ 2 (x)n+1
* si n est pair (2)
n + 1 12n+1

|𝑇𝑛(𝑥)| < 10−4


156
Sachant que sup|x| =
21
(1) ∶ n = 2k + 1 (∀k ≥ 0); |rn | = 0 < 10−4
n+1
2*(156⁄21)
(2) ∶ n = 2k + 1 (∀k ≥ 0); |rn | = 0 < 10−4
(n+1)*12n+1

Pour n = 0, |r0 | = 1,23804523 > 10−4


.
.
.
Pour n = 6, |r0 | = 0,00954062 > 10−4
Pour n = 16, |r0 | = 0,00003323 < 10−4
Le degre minimal de Tn(x) est de n = 16.
𝑄𝑈𝐸𝑆𝑇𝐼𝑂𝑁 5
.𝑎) 𝐷𝑒𝑡𝑒𝑟𝑚𝑖𝑛𝑜𝑛𝑠 𝑠𝑖 𝑓 𝑒𝑠𝑡 𝑐𝑜𝑛𝑡𝑖𝑛𝑢𝑒 𝑒𝑛 𝑥 = 0
.𝐷𝑓 = 𝑅, 0 ∈ 𝑅 (1)
.𝐹(0) = 0 (2)

11⁄ )2
lim 𝑓(𝑥) = lim (𝑒)−( 𝑥
𝑥→0 𝑥→0
11⁄ )2
.Posons y = lim (𝑒)−( 𝑥
𝑥→0
11⁄ )2
.On a ∶ ln 𝑦 = ln(𝑒)−( 𝑥

−𝑥
ln 𝑦 = ( )2
𝑥

ln 𝑦 = −∞
.D’ou y = 0 car lim 𝑙𝑛 𝑦 = −∞
𝑦→0

Ainsi lim 𝑓(𝑥) = 0 (3)


𝑥→0

.Par definition de la continuite on a ∶


0 ϵ Df D′apres (1)
{ lim 𝑓(𝑥) = 𝑓(0) = 0 𝐷′𝑎𝑝𝑟𝑒𝑠 (2) 𝑒𝑡 (3)
𝑥→0

.𝐷′𝑜𝑢 𝑓 𝑒𝑠𝑡 𝑐𝑜𝑛𝑡𝑖𝑛𝑢𝑒 𝑎𝑢 𝑝𝑜𝑖𝑛𝑡 𝑥 = 0


𝑏)𝑃𝑎𝑟 𝑟𝑒𝑐𝑢𝑟𝑟𝑒𝑛𝑐𝑒, 𝑚𝑜𝑛𝑡𝑟𝑜𝑛𝑠 𝑞𝑢𝑒 𝑝𝑜𝑢𝑟 𝑡𝑜𝑢𝑡 𝑛 ≥ 0; 𝑝𝑜𝑢𝑡 𝑡𝑜𝑢𝑡 𝑥 ≠
0;
1 11 11⁄ )2
𝑄(𝑛) :𝑓 (𝑛) (x)= . ( )−(
𝑛 .𝑃3𝑛 ( ). 𝑒
𝑥 , 𝑎𝑣𝑒𝑐 𝑃3𝑛 . 𝑢𝑛 𝑝𝑜𝑙𝑦𝑛𝑜𝑚𝑒.
11 𝑥

.𝑈𝑡𝑖𝑙𝑖𝑠𝑎𝑡𝑖𝑜𝑛:

11⁄ )2
𝑃𝑜𝑢𝑟 𝑛 = 0, 𝑜𝑛 𝑎 ∶ 𝑓 (0) (𝑥) = 𝑓(𝑥) = (𝑒)−( 𝑥 =𝐶∗
11 2
(𝑒)−( ⁄𝑥) , 𝐶 = 𝐶𝑜𝑛𝑠𝑡𝑎𝑛𝑡𝑒 = 1.
1 11 11⁄ )2 11 11⁄ )2 11⁄ )2
. ( )−(
0 .𝑃0 ( ). 𝑒
𝑥 =𝑃 . ( ).
0 𝑥 (𝑒)−( 𝑥 = 𝐶 ∗ (𝑒)−( 𝑥 ,𝑃 .
0 ∗
11 𝑥
11
( ) = 𝐶, 𝐶𝑡𝑒.
𝑥

.𝐷′𝑜𝑢𝑄(0) = 0.

𝐻𝑒𝑟𝑒𝑑𝑖𝑡𝑒: 𝑠𝑜𝑖𝑡 𝑛 ≥ 0, 𝑥 ≠ 0,
𝑠𝑢𝑝𝑝𝑜𝑠𝑜𝑛𝑠 𝑞𝑢𝑒 𝑄(𝑛) 𝑠𝑜𝑖𝑡 𝑣𝑟𝑎𝑖𝑒 𝑒𝑡 𝑚𝑜𝑛𝑜𝑡𝑜𝑛𝑒 𝑄(𝑛+1)
𝐷𝑜𝑛𝑐, ∀𝑛 ≥ 0, 𝑥 ≠ 0,
1 11 11 2
𝑓 (𝑛+1) (x)= (𝑛+1) .𝑃3(𝑛+1). ( ). (𝑒)−( ⁄𝑥)
11 𝑥
𝐸𝑛 𝑒𝑓𝑓𝑒𝑡:
1 11 11 2 ′
𝑓 (𝑛+1)
(x)=[𝑓 (𝑛) ′
(x)] =[ . 𝑃 . ( ). (𝑒 )−( ⁄𝑥) ]
11(𝑛) 3𝑛 𝑥
1 11 11 2
= [𝑃3𝑛. ′ . ( ). (𝑒 )−( ⁄𝑥) +
11𝑛 𝑥
11 242 11 2
𝑃3𝑛. ( ). . (𝑒 )−( ⁄𝑥) ]
𝑥 𝑥3
1 11 242 11 11⁄ )2
= [𝑃3𝑛. ′ . ( ) + . 𝑃3𝑛. ( )]. (𝑒)−( 𝑥
11𝑛 𝑥 𝑥3 𝑥
1 11 11 11⁄ )2
= . . ( )−(
𝑛 [𝑃3𝑛+1 ( ) + 242. 𝑃3𝑛+3 ( )]. 𝑒
𝑥
11 𝑥 𝑥

𝑎 𝑎 1 1 1 1 1
𝑂𝑛 𝑎: 𝑃𝑛 ( )′ = 𝑃𝑛+1 ( ) ; . 𝑃 ( )= 𝑃𝑚+𝑛 (𝑥); a. 𝑃𝑛 (𝑥)= 𝑃𝑛 (𝑥)
𝑥 𝑥 𝑥𝑛 𝑛 𝑥
.𝐸𝑡 𝑒𝑛𝑓𝑖𝑛

1 1 1
𝑃𝑛 ( ) +𝑃𝑚 ( )= 𝑃𝑛 ( )
𝑥 𝑥 𝑥

.𝐴𝑖𝑛𝑠𝑖 𝑜𝑛 𝑎:
1 11 11
𝑓 (𝑛+1) (x)= 𝑛 .[ 𝑃3𝑛+1 ( ) + 𝑃3𝑛+3 ( )].
. .
11 𝑥 𝑥
11 2
(𝑒 )−( ⁄𝑥)

1 11 11 2
= .[ 𝑃3𝑛+3. ( )]. (𝑒)−( ⁄𝑥) =
11𝑛 𝑥
1 11 11 11 2
. . 𝑃3(𝑛+1). ( ). (𝑒)−( ⁄𝑥)
11𝑛 11 𝑥

1 11 11 2
= . 11. 𝑃3(𝑛+1). ( ). (𝑒)−( ⁄𝑥)
11𝑛+1 𝑥

𝐴𝑖𝑛𝑠𝑖: 𝑓 (𝑛+1) (x)


1 11 11⁄ )2
= . 11. 𝑃 3(𝑛+1). ( ). (𝑒)−( 𝑥
11𝑛+1 𝑥

𝐷′𝑜𝑢 𝑄(𝑛+1)
1 11 11⁄ )2
𝐶𝑜𝑛𝑐𝑙𝑢𝑠𝑖𝑜𝑛: ∀𝑛 ≥ 0, ∀ 𝑥 ≠ 0; 𝑓 (𝑛) (𝑥) = ( )−(
𝑛 . 11. 𝑃3𝑛 ( ). 𝑒
. 𝑥
11 𝑥

𝑐) 𝑓 (𝑛) (0) = ?
(𝑛) 1 11 11⁄ )2 1 11
𝑂𝑛 𝑎 ∶ 𝑓 (𝑥) = 𝑛.𝑃 ( ). (𝑒)−(
3𝑛.
𝑥 = 𝑛 . 𝑃3𝑛 ( ). 𝑓(𝑥)
.
11 𝑥 11 𝑥
(𝑛) 1 11
∀ 𝑥 ≠ 0; 𝑓 (0) = . 𝑃3𝑛. ( ). 𝑓(0) = 0
11𝑛 𝑥
(𝑛)
𝐷′𝑜𝑢 𝑝𝑜𝑢𝑟 𝑡𝑜𝑢𝑡 𝑛 ≥ 1 , ; 𝑓 (0) = 0

.𝑆𝑒𝑟𝑖𝑒 𝑑𝑒 𝑀𝑎𝑐 𝐿𝑎𝑢𝑟𝑖𝑛:



𝑓 (𝑘) (0). 𝑥 𝑘
𝑇(𝑥) = ∑ = 0 𝑐𝑎𝑟 𝑓 (𝑘) (0) = 0
𝑘!
𝑘=1

𝑑) 𝑓(𝑥) = 𝑇(𝑥) = 0 𝑝𝑜𝑢𝑟 𝑥 = 0


. 1 2
1 -( )
P3(n+1) *( )*e 𝜀 *xn+1
𝜀
Rn(x) = (n+1)!
. 1 2
1 -( )
P3(n+1) *( )*e 𝜀 *xn+1
𝜀
lim Rn = lim (n+1)!
n→∞ n→∞

1 2
1 -(𝜀) xn+1
= P3(n+1) * ( ) *e lim
𝜀 n→∞ (n+1)!

0 si |x| ≤ 1
={
∞ si |x| > 1
0 si |x| ≤ 1
Pour dire lim Rn = {
n→∞ ∞ si |x| > 1
Sachant que ∀n ≥ 1 T(x) = 0 = f(0), prenons la valeur unique de x
= 0 qui fait que f(x) = T(x) pour x = 0.

You might also like